HELP NUMBER 10 please!!!

HELP NUMBER 10 Please!!!

Answers

Answer 1

Note:

0.13 x 16500 = 2145

16500 - 2145 = 14355 for the first year.

Continue this for 7 more years to get your answer.


Related Questions

Which rational number equals 0 point 2 with bar over 2? (1 point)

Group of answer choices

1 over 10

2 over 10

2 over 9

1 over 9

Answers

Answer:

1 over 10

Step-by-step explanation:

I guessed

1 out 10 because you can divide both side by the number

Just the last question thank you!!

Answers

Answer:

6 X 10^21

Step-by-step explanation:

PLEASEEE HELP TOMORROW IS THE LAST DAY TO THEN STUFF IN AND ITS REPORT CARDS

Answers

This is ezz homework pay attention in class

I think this is correct

Find m 1 if m 2=47.6

Answers

Answer:

try 130.4, I got my answer by subtracting 47.6 with 180. How'd I get 180? the angles are obtuse so that means it'll add up to 180. Thus the answer is 130.4

Plz help me I’ll give brainliest

Answers

Answer:

it is A i could be wrong

Step-by-step explanation:

Tickets that cost $88 for a professional women’s championship soccer game are offered at a 15% discount. What is the sale price?

Answers

Answer:

$74.80

Step-by-step explanation:

10% = 8.8

5% = 4.4

8.8 + 4.4 = 13.2

88 - 13.2 =$74.80

EVALUATE
9(6−2.4)−10

Answers

Answer:

The answer is 22.4.

Answer:

22.4

Step-by-step explanation:

you multiply 6 and -2.4 by 9

that gives you 54 and -21.6

then you add -10 to -21.6 giving you -31.6

lastly subtract -31.6 out of 54

which gives you 22.4

I chose the wrong screensMaggie is converting 10,560 feet to miles by using StartFraction 10,560 feet Over 1 EndFraction times StartFraction 1 mile Over 5,280 feet EndFraction. What should be Maggie’s answer?hots for the last one, sorry about that!

Answers

5,280 Maggie’s answer I guess not sure

How do I find x on lines​

Answers

The center line that goes from left to right is your x axis and the line that goes from top to bottom is you y axis

A racing car can complete an 700 m long lap in 14 s. What is the speed of the
car?

Answers

Answer:

[tex]\boxed {\tt 50 \ m/s}[/tex]

Step-by-step explanation:

To find the speed, divide the distance by the time. We can use the following formula:

[tex]s=\frac{d}{t}[/tex]

The racing car completed a 700 meter lap in 14 seconds. So, the distance is 700 meters and the time is 14 seconds.

[tex]d= 700 \ m \\t= 14 \ s[/tex]

Substitute the values into the speed formula.

[tex]s=\frac{700 \ m }{14 \ s}[/tex]

Divide.

[tex]s= 50 \ m/s[/tex]

The speed of the racing car is 50 meters per second

Answer:

The answer is 50 m/s

Step-by-step explanation:

The speed of an object can be found by using the formula

[tex]v = \frac{d}{t} \\ [/tex]

where

v is the speed

d is the distance

t is the time

From the question

d = 700 m

t = 14 s

We have

[tex]v = \frac{700}{14} \\ [/tex]

We have the final answer as

50 m/s

Hope this helps you

A music app charges $2 to download the app plus $1.29 per song download, Identify the rate of change.

Answers

Answer: 1.29

Step-by-step explanation: y=1.29x+2

1.29 is the coefficient of X, slope, and the rate of change.

A bird leaves it's nest and travels 15 miles per hour downwind for x hours. On the return trip, the bird travels 3 miles per hour slower and has 2 miles left after x hours. A. What is the distance of the entire trip? ___ Miles B. How long does the entire trip take? __hours ___minutes and ___ seconds

Answers

Answer:

Step-by-step explanation:

Distance = Speed * time

If a bird leaves it's nest and travels 15 miles per hour downwind for x hours, the distance covered will be:

D = 15x

x = D/15 ....... 1

If on the return trip, the bird travels 3 miles per hour slower and has 2 miles left after x hours

Her speed = 15 - 3 = 12mi/hour

If she has 2 miles left after x hours, new distance will be D-2, using the formula for distance:

D+2 = 12x

x = D-2/12  ....... 2

Equate 1 and 2:

D/15 = D-2/12

Cross multiply

12D = 15(D-2)

12D = 15D - 30

12D-15D = -30

-3D = -30

D = 10 miles

The distance for the entire trip will be D + (D-2) = 10 + (10-2)

= 10 + 8

= 18 miles

To get the time x, we will substitute D = 10 into equation1

D = 15x

10 = 15x

x = 10/15

To minutes

x = 10/15 * 60

x = 40 minutes

For the going trip

Time taken = x hours

Return trip = x hours

Total time for the entire trip = x+x = 2x

= 2(40)

= 80 minutes

Hence the total time taken for the entire trio will be 1 hour 20minutes

find the area of the equilateral triangle.if its perimeter is 24cm​

Answers

Answer:

16√3 cm²

Step-by-step explanation:

The perimeter of a triangle is the sum of its all three sides. Since this is an equilateral triangle, all sides are equal.

Let's consider one side of the triangle to be 'x'

Givent that, the perimeter is 24cm,

The equation should be x + x + x = 24

⇒3x = 24

∴ x = 8 cm

To find the area of the triangle, we need to find the height, and for that, we can use trigonometry.

Since it is an equilateral triangle, all angles are exact 60°.

let's draw a line and mark it as 'h'.

we can use sine formula to find out the opposite i.e. h

sin∅ = opposite ÷ hypotaneous

sin 60° = h ÷ 8

h = 8 sin 60°

h= 4√3

Now, let's find the area

Area = 1/2 × base × height

Area = 1/2 × 8 × 4√3

area= 16√3 cm²

Evaluate the following expression. $$(10 + 2) \div 6 + 5^3$$

Answers

Answer:5

Step-by-step explanation:

If a recipe calls for 4 1/2 cups of flour for 48 cookies, how much flour would be needed for 24 cookies?

Answers

Answer:

2 1/4 is the answer

Step-by-step explanation:

4 2/4÷2= 2 1/4

Answer:

2 1/4

Step-by-step explanation:

Set up a table with the labels flour and cookies on the flour side have 4 1/2 on the cookies side have 48 then in another row 24. Divide 24 by 48 to find out what to multiply 4 1/2 by to get the answer.48 divided by 24 is .5 so you would multiply 4 1/2 by .5 to get 2 1/4

How does the Electoral College work?
A
It guarantees that the winner of the popular vote wins the presidency.
B
The Electoral College casts its votes on the first Tuesday in November.
The Electoral College casts its votes on January 20.
D
Each state casts a number of electoral votes equal to their members of Congress.

Answers

Answer:

D

Step-by-step explanation:

The answer is D.

Hope this helps:)

What is the slope of the line y = -2?

Answers

Answer:

0

Step-by-step explanation:

There is no x, therefore there is no slope.

Lexi runs the snack stand at Belleview Community Pool. This morning, she withdrew $150 from the stand's bank account to buy supplies. When the stand closed this evening, she deposited $437 from the day's sales. What was the total change in the stand's bank account relative to where it started?

Answers

Answer:

Step-by-step explanation:

Let the initial money in the account be y.

Lexi withdrew $150 from the stand's bank account to buy supplies. This will be:

= y - $150

Later, she deposited $437 from the day's sales. The money in the account will now be:

= y - $150 + $437

= y + $287

The total change in the stand's bank account relative to where it started will be the initial amount + $287.

Answer:

287

Step-by-step explanation:

Seth is trying to pay off his credit card. He plans to pay it off in 24 months. Interest on the card is compounded monthly at a fixed annual rate of 11.6%. Seth currently has a balance of $18,875 on his credit card. Use the formula for the sum of a finite geometric series to determine Seth’s approximate monthly payment.


A.

Seth’s approximate monthly payment will be $753.97.

B.

Seth’s approximate monthly payment will be $834.75.

C.

Seth’s approximate monthly payment will be $884.99.

D.

Seth’s approximate monthly payment will be $868.35.

Answers

Answer:

$884.99

Step-by-step explanation:

Just took the quiz and this was the right answer. :)

Answer:

884.99

Step-by-step explanation:

is this a function or not

Answers

Answer:

I believe it is

Step-by-step explanation:

None of the x and y values have the same number

Answer:

It is not the input 5 has 2 different output values, same as the input 3 it has more than one different output

Step-by-step explanation:

Hope this helps!

• Lawson already has $60 in his wallet but needs a total of at least $500 for his holiday. He gets
paid $30 per day for delivering newspapers. What is the least number of days he must work to
get enough money for his holiday?

Answers

Answer:

15

Step-by-step explanation:

500-60=440

440/30=14.66666667

simplify 14.66666667= 15

             

Simplify.

8200−−−√−3162−−−√

Answers

Answer:

the question is supposed to be [tex]8 \sqrt 200 - 3\sqrt 162\\[/tex]

i think the answer is [tex]53\sqrt 2[/tex]

I'll update if its right or wrong

Step-by-step explanation:

The expression 8√200 − 3√162 in the simplified form will be 53√2.

What is Algebra?

The analysis of mathematical representations is algebra, and the handling of those symbols is logic.

PEMDAS rule means the Parenthesis, Exponent, Multiplication, Division, Addition, and Subtraction. This rule is used to solve the equation in a proper and correct manner.

The expression is given below.

⇒ 8√200 − 3√162

Simplify the expression, then we have

⇒ 8√200 − 3√162

⇒ 80√2 − 27√2

⇒ 53√2

The expression 8√200 − 3√162 in the simplified form will be 53√2.

More about the Algebra link is given below.

https://brainly.com/question/953809

#SPJ2

In a bag full of beans and rice, the ratio by weight of beans to rice is 2 to 9. If the bag has a total weight of 3 kilograms, how many kilograms of beans are there in the bag? (round to nearest thousandths)

Answers

Answer: 0.667 kilograms

Step-by-step explanation:

From the question, we are informed that in a bag full of beans and rice, the ratio by weight of beans to rice is 2 to 9.

If the bag has a total weight of 3 kilograms, the amount of kilograms of beans that are there in the bag will be:

= 3/9 × 2 kilograms

= 6/9 kilogram

= 0.667 kilograms

Monique purchases a $5,100 dining room set. She can't afford to pay cash, so she uses the installment plan, which requires an 18% down payment. How much is the down payment?​

Answers

Answer:

$918 is her down payment

Step-by-step explanation:

.18×5100

The Down payment is $918.

What is down payment?

Down payment is the initial amount which is paid by the person who purchases an item on credit.

It is the advanced payment which is given at starting of purchase an item. It is a cash payment which is made at the beginning of a purchase of an item.

As per the question, monique purchased $5100 dining room set.

As she cannot afford to pay all amount immediately, she took it on credit on installment but she has to pay down payment, an initial amount of certain percentage, which is 18% down payment.

Cost of an dinning room set = $5100

Down payment = 18%

So, the amount of down payment, which she will pay intially is $5100 × 18/100 = $918.

Monique will pay $918 as a down payment.

$918 is the advanced amount which will be paid by monique to purchase dining room set as a down payment.

To know more about down payment click the link below

https://brainly.com/question/1698287

#SPJ2

A frozen dinner at a grocery store is purchased for $2.20 wholesale and is marked up by 40 percent. A customer wants to buy four frozen dinners. How much will the customer pay, including paying 9 percent sales tax? Round your answer to the nearest cent. $8.80 $10.07 $13.43 $14.39

Answers

Answer:

$13.43

Step-by-step explanation:

Given that :

Price of dinner wholesale = $2.20

Markup % = 40%

Hence, selling price = (100 + 40)% * $2.20

Selling price = 140% * $2.20

Selling price = 1.4 * $2. 20 = $3. 08

Hence, price of 4 frozen dinner :

$3. 08 * 4 = $12.32

9% sales tax:

9% * $12.32

0.09 * $12.32 = 1.1088

Total amount paid :

$(12.32 +. 1.1088)

= $13.4288

= $13.43

Answer:

The answer is $13.43

Step-by-step explanation:

I just answered that question, and got it right

I need help please it’s a test and I don’t understand

Answers

Answer:

to get x look at that triangle that it is in I know the angles in a triangle add up to 180° so it will be 180 - 63 degrees plus 36 degrees then to get z it will be 36 + 63 because interior angles add up to one exterior angle then to get y addz + 13 degrees then subtract it from 180°

Anup bought 50 glasses
for Rs 1800
out of which 4 were broken. If
he sold the rest of the
glasses for Rs. 35 each. How much
profit or loss percentage did
he bear?​

Answers

Step-by-step explanation:

50 glasses for Rs 1800

4 broken

sold each for Rs.35 each

50-4=46

35*46=1610

Determine which quadrant the point is in: (-3,-4)
Quadrant

Answers

the answer is quadrant 3

Answer:

Its in quadrant 3

Step-by-step explanation:

Since they are both negative they go into the third quadrant

I’ll give brainley and 10 points

Answers

Answer:At a charity fund raiser , adult tickets were sold for 8$ children each and children’s tickets were sold at 2$ each. write an algebraic expression for the total amount raised from the sale of tickets. At a charity fund raiser , adult tickets were sold for 8$ children each and children’s tickets were sold at 2$ each. write an algebraic expression for the total amount raised from the sale of tickets. How much money was raised if the fundraiser sold 238 adult tickets and 375 children’s tickets?

Step-by-step explanation:

2,654$

Step-by-step explanation:

8a + 2c

8 x 238 = 1,904

2 x 375 =  750

750 + 1,904 = 2,654

2,654$

Answer:At a charity fund raiser , adult tickets were sold for 8$ children each and children’s tickets were sold at 2$ each. write an algebraic expression for the total amount raised from the sale of tickets. At a charity fund raiser , adult tickets were sold for 8$ children each and children’s tickets were sold at 2$ each. write an algebraic expression for the total amount raised from the sale of tickets. How much money was raised if the fundraiser sold 238 adult tickets and 375 children’s tickets?

Step-by-step explanation:

2,654$

Step-by-step explanation:

8a + 2c

8 x 238 = 1,904

2 x 375 =  750

750 + 1,904 = 2,654

2,654$:

​ Select all the ratios that represent unit rates.
A.
7 days
1 book

B.
15 gallons
350 miles

C.
1 year
$30,000

D.
1 minute
90 words

E.
5 students
5 teachers

Answers

Answer:

The answers to this problem are B, D, and A.

Step-by-step explanation:

I just did the problem. I always get 100% on these quizzes.

Other Questions
After a virus uses a host cell, the host cell oftenis damaged or destroyeddevelops a capsidis no different than beforeis refreshed and healthy Which one of the following words is a pronoun A) Two B) Him C) Boys D) Go Authors provide context for us to make conclusions. What is context?A.factsB.implied meaningsC.facts and previous knowledgeD.the circumstances that make up a situation Banana Computer Company sells Banana Computers both in the domestic and foreign markets. Because of the differences in the power supplies, a Banana computer purchased in one market cannot be used in the other market. This means that the company can use third degree price discrimination in order to maximize profits. Lets suppose that it costs $1,000 to produce each computer (this is marginal and average cost). Lets suppose further that the domestic and foreign demand curves are given as follows (the subscript "F" denotes "foreign" while the subscript "D" is used to denote "domestic"):PD=13,000 -20QDPF= 17,000-40QFRequired:a. What prices maximize profits for this firm? How many computers do they sell in each market? How much profit does the company earn? b. Now, suppose that somebody figured out a wiring trick that allows a Banana computer built for either market to be costlessly converted so that it works in the other market. This destroys the company's ability to practice third degree price discrimination and forces them to charge the same price in both markets. What price maximizes the company's profits now? How many computers will they sell in each location? How much profit does the company earn? Which equation shows a proportional relationship between x and y? an angles measure is 9 degrees more than 2 times the measure of its supplement. Find the measure of the angle Please help me i cant get this one right. someone help me please if you know how to do this. HELP HELP!!! ASAAAAPPPPP Which examples are compound sentences? Select three options 1. When you write HTML code, you use ______ to describe the structure of information on a webpage. a. a web address b. tags c. styles d. links Margie order 120 pizzas for the school pizza party one tenth of the pizzas were cheese how many cheese pizzas she ordered 1. The human body maintains a constant internal temperature of 98.2F - 98.8F What changes occurwhen the hypothalamus detects a temperature of 95F? Is this a negative or positive feedback? they were a south american indian culture that was devestated by pizarro and the spanish in the 1530s describe and explain how peristalsis helps ureter to do its function. Jessica is traveling from Miami, Florida, to Chicago, Illinois. Using the map, tell one way the land will change during the second half of her trip. a. The elevation will drop below sea level.b. The elevation will gradually decrease.c. The elevation will gradually increase.d. The elevation will remain the same. Who is you're least favorite character in the outsiders novel and why ? can someone please help me? plz help and plz show work 10. Simplify 5-3y+(-9y)+5.5 Check all that apply: functions of the skeletal systema. storesb. protectsc. supportsd. movese. produces blood cellsf. provides are for muscle attachment